Differential Equations, Preliminary Edition. By Blanchard, Devaney, Hall. Published by PWS Publishing Co., Boston, MA, 1995.

Known "bugs" in the Preliminary Edition

Blinks indicate corrections that should be made.

Section 1.1, Page 11.

At the bottom of the page, the second to last line of text should read:

this case, eating rabbits helps the foxes, so we add a term of the form deltaRF.


Section 1.2, Page 28.

In the middle of the page, in the calculation of the constant c_3, there is an error in the approximation of e^(0.05*10). The value listed there, 1.0513, is incorrect. The correct value should be 1.6487. Even though we botched this calculation, we did manage to get the rest of the problem right and save ourselves from bouncing any checks.


Section 1.3, Page 40 and Figure 1.23

The differential equation at the bottom of the page is not the one used in the previous section on page 30. The original equation is

dS/dt = (2000 - 3S)/100.
The same problem occurs with the figure. It has been suggested by the authors that this error occured as a result of the picture looking better with the new numbers, but this has not been confirmed.


Section 1.3, Page 44, Figure 1.28

In the text accompanying Figure 1.28, replace K = 1 with K = 2.


Section 1.3, Page 45, Exercise 1.
The answer in the back of the book is wrong. The sketch provided there is apparently for

dy/dt = t - t^2.

For the correct slope field, click here .


Section 1.4, Page 59, Exercise 11.
The hint included with the answer in the back of the book suggests that you recall the Existence and Uniqueness Theorem. This is a little difficult since this Theorem appears for the first time in the next section.

The solution should read: From qualitative analysis, solutions with initial condition w(0) = 0 should increase until reaching the equilibrium solution at
w = 3. However, the numerical solution indicates that the solution oscillates about w = 3.


Section 1.4, Page 60, Exercise 19.
The answer in the back of the book for part (c) is incorrect. Replace

0.2.115 with 2.115 .


Section 1.5, Page 67, Second line of second paragraph.
The line should read: "It is reasonable to guess that, as t approaches positive infinity, y(t) approaches -1."


Section 1.5, Page 69, Exercise 9.
Although this is one of our favorite problems, the astute mathematician will notice that even though y(t) is a solution to the differential equation, it may not be defined for all time and hence the statement y(t) < 1 + t^2 for all t is inaccurate. One can correct this technicality by realizing that y_2(t)=t^2 is also a solution to the ode.


Section 1.6, Page 74, Figure 1.52(a). The lower equilibrium point is labeled incorrectly. y = 0 should read y = -3.


Section 1.6, Page 84. The displayed equation in the middle of the page
dS/dt = kS (1 - S/M) (S/M - 1) should read

dS/dt = kS (1 - S/N) (S/M - 1).


Section 1.6, Page 85. In the caption of Figure 1.75, the displayed equation should read

dS/dt = kS (1 - S/N) (S/M - 1).


Section 1.7, Page 93. In the last line of text, there is an "= 0" at the end of the sentence. Please delete this. The line should read: "Figure 1.81 shows the bifurcation diagram for f_mu(y) = y^2 - 2y + mu."


Section 1.7, Page 94. The beginning of the second paragraph should read:

First, if alpha < 0, the term ...

Also, the caption in Figure 1.82 is incorrect. The second sentence should read:

Note that for alpha < or = 0 the graph crosses the y-axis once, whereas if alpha > 0, the graph crosses the y-axis three times.


Section 1.7, Page 97, Figures 1.86 and 1.87. These figures do not correspond well with the text that describes them on pages 96 and 97. In Figure 1.86, the bottom curve should have three real roots rather than just one implying three equilibrium points. But this only happens for -4/27 < mu < 0. We chose a mu value too negative or too far away from the bifurcation point mu = 0!

In Figure 1.87, the top curve should have three real roots rather than just one implying three equilibrium points. Again, this happens for -4/27 < mu < 0. When graphing the top curve, we overshot the value mu = 0, and actually chose a positive mu value!

The moral of this story is that bifurcations should be analyzed locally. When a bifurcation happens at say mu = 0, then one should really only look at graphs with mu values very close to 0. We failed to do this and obtained misleading graphs.


Section 1.7, Page 100, Exercise 3. The answer in the back of the book is incomplete. In addition to the bifurcation at a = 2, there is a similar bifurcation at a = -2. Here, the phase lines for a > -2, a = -2 and a < -2 are qualitatively the same as the ones for a < 2, a = 2 and a > 2 respectively.


Section 1.8, Page 113, Exercise 25. The answer in the back of the book is incorrect. In our haste, we failed to express the amount of dioxin as a concentration. The correct answer is

1700/400 = 4.25 ppb .

Note that starting with a concentration of 2 ppb and adding water with a concentration of 5 ppb will result in a concentration somewhere between 2 and 5, nowhere near 1700! (Our grad students must have been sleeping on that one.) Although the problem is solvable without converting ppb by weight to lbs/gal, we recommend this approach to clarify the problem.


Section 1.9, Page 124, Exercises 5-7. Change the words "the phase plane" to "the phase line."
The answers in the back of the book are also incorrect. The pictures should be phase lines not slope fields as currently shown.


Section 1.9, Page 125, Exercise 15. This problem and the answer in the back of the book are messed up. As stated in the problem, the volume in the tank remains constant, but the answer in the back of the book states that the volume is given by 10 + 3t. We suggest replacing the problem with the following new problem:

Consider a very large vat that initially contains 10 gallons of clean water. Suppose water starts entering the vat from two pipes. From the first pipe, saltwater containing 2 pounds of salt per gallon enters the vat at a rate of 1 gallon per minute. From the second pipe, saltwater containing 0.2 pounds of salt per gallon enters the vat at a rate of 5 gallons per minute. Suppose the liquid is kept well mixed and saltwater is removed from the vat at a rate of 3 gallons per minute.

With these modifications, the answer in the back of the book is now correct.


Lab 1.1, Page 129.
The astute observer and well-traveled person might have noticed that Alaska has nowhere near 4 million people in its current population. The list of populations for Alaska is really that of Alabama . Sorry for offending all you "Northern Exposure" fans.


Section 2.1, Page 145, Exercise 13.
The instructions and answer are incorrect. In order to have five times as many rabbits(prey) as fox (predator) it is necessary that R>5F not 5R>F as is currently in the instructions. Also, the answer should read: "In both systems, make

dF/dt = - F +0.9 FR +k (R-5F)
where k is an immigration rate parameter."


Section 2.4, Page 180,
The second equation on this page should read

d^2y_1 / dt^2 = ... = -3y_1.


Section 2.4, Page 183, Exercise 13.
There is a slight error in the back of the book. In the equation for dv/dt, the correct coefficient for the y term is -(k_1 + k_2)/m.


Lab 2.3, Page 221.
There is an error in the first table. The first entry in the column for y_2(t) should be 0.74, not .074.


Section 3.1, Page 242, Exercise 15.
In the statement of the problem, the determinant condition should read

det A = 0.


Section 3.2, Page 247.
Three lines from the bottom of the page, Y_2(t) should be replaced by Y_1(t).


Section 3.2, Page 256.
In the Theorem at the top of the page, when there are distinct, real eigenvalues lambda_1 and lambda_2, the solutions are

Y_1(t) = e^{lambda_1 t} V_1 and Y_2(t) = e^{lambda _2 t} V_2.


Section 3.2, Page 261, Exercises 23-28.
The instructions next to the third bullet are vague. Feel free to omit these when doing problems 23-28.


Section 3.2, Page 261, Exercise 27.
There is a typo in the answer to part c in the back of the book (p. 621). The term 2 + sqrt{33} should be replaced by simply 2 sqrt{33}.


Section 3.3, Page 271.
On line 5 and line 11, change V_2 to V_1. Also, the eigenvalue on line 6 should be

(-3 + 5^{1/2})/2.


Section 3.3, Page 276, Exercise 15d.
The y(t)-graph in the answer in the back of the book is wrong. We'll put the correct one up here soon. Also, although the answer provides a v(t)-graph, this was not called for in the problem. Nice of us to give you extra info, ehh?


Section 3.4, Page 290, Exercise 9b.
The answer in the back of the book is wrong. It should read

Y(t) = ( cos 2 t, -sin 2t)


Section 3.4, Page 290, Exercise 11a.
The answer in the back of the book is wrong. The e^{4t} term should be placed between k_2 and the given vector.


Section 3.4, Page 291, Exercise 15.
The answer in the back of the book asserts that graphs 2 and 5 are correct, and that the natural period of graph 1 is not constant over time. This is somewhat difficult to see, so one might argue that graph 1 is also acceptable as the graph of the x-coordinate of a solution.
Also, in the exercise, graphs 1, 4, and 6 have axes labled y instead of x. This is a typo.


Section 3.4, Page 292, Exercise 22.
The hint given in the statement of the problem is incorrect; k_1 and k_2 should be interchanged.


Section 3.4, Page 292, Exercise 25.
The answer in the back of the book is incorrect. Part (c) should not have a graph at all. It should say underdamped . Part (d) should have the graph from part (c). Part (e), the general solution, should read:

Y(t) = k_1 e^{-t/2}( 2cos (sqrt{15}t/2), -cos (sqrt{15}t/2) -sqrt{15} sin (sqrt{15}t/2)) +

k_2 e^{-t/2}( 2sin (sqrt{15}t/2), sqrt{15} cos (sqrt{15}t/2) -sin (sqrt{15}t/2))


Section 3.6, Page 311.
As it reads, we have "imaginary" periods for our spiral sinks in the middle of the page. There are two places where this happens. The natural period 2 PI/ sqrt{1-2a} should be replaced by

2 PI/ sqrt{2a-1}
in both cases.


Section 3.6, Page 316.
As it reads, we have an "imaginary" period for our underdamped oscillator at the top of the page. The period 4 PI/ sqrt{K_d^2 - 4} should be replaced by

4 PI/ sqrt{4 - K_d^2}.


Section 3.6, Page 320, Exercise 9.
The answer in the back of the book is incorrect. Part (a) should read:

Y(t) = k_1 e^{-3t/2}( 8cos (sqrt{79}t/2),-cos (sqrt{79}t/2) -sqrt{79} sin (sqrt{79}t/2)) +

k_2 e^{-3t/2}( 8sin (sqrt{79}t/2), +sqrt{79} cos (sqrt{79}t/2) -sin (sqrt{79}t/2))

Part (b) should read:

Y(t) = e^{-3t/2}( 2cos (sqrt{79}t/2)+(10sqrt{79} /79) sin (sqrt{79}t/2),

cos (sqrt{79}t/2) -(21sqrt{79} /79) sin (sqrt{79}t/2) )


Section 3.6, Page 321, Exercise 27.
The answer in the back of the book is incorrect. In parts (a) and (b), there should be sqrt signs over 3t/2 and 3/2 everywhere they occur in the answers. Moreover, in part (b) the coefficients of the sin terms are incorrect. Both coefficients should be replaced by -sqrt{6}.


Section 3.6, Page 323, Exercise 35.
The answer in the back of the book is incorrect. Part(a) should read:

md^2y/dt^2 - k_{mf}dy/dt + k_{s}y = 0

and

dy/dt=v, dv/dt=-(k_s/m)y + (k_{mf}/m)v

Part (b) should be adjusted to: Modify the part of this section entitled "The Harmonic Oscillator" to consider the change in the sign of the dy/dt term.


Section 3.7, Page 331-2,
The equation at the bottom of the page is wrong. It should read

dx/dt = 0.1x - y
dy/dt = x + 0.1 y
dz/dt = -0.2z

Also, on the following page, the eigenvalues in the first line should be 0.1 + i and 0.1 - i.


Section 3.7, Page 335,
The authors have a little trouble with decimals every now and then. In lines 10 and 12, the eigenvalue should be lambda_1 = 0.2 .

The equations for the eigenvector for lambda_1 = 0.2 should be

AY_1 = 0.2 Y_1

or

y_1 = 2 x_1
2 z_1 = 2 y_1
4 x_1 = 2 z_1

The equations for the eigenvector for lambda_2 = -0.1 + isqrt{0.03} should be

y_2 = (-1 + i sqrt{3}) x_2
2 z_2 = (-1 + i sqrt{3}) y_2
4 x_2 = (-1 + i sqrt{3}) z_2


Section 3.7, Page 338, Exercise 11a.
Somehow a negative sign emerged in the answer. The correct eigenvalues are -2 and 1, not -2 and -1.


Section 4.1, Page 359, Exercise 2
The third system in this problem actually doesn't have an equilibrium point at the origin. Change the second equation in this system to read dy/dt = 4x + 3 cos y - 3.


Section 4.2, Page 373
We have inadvertantly introduced the variable y here in the text when we really meant to use theta. This occurs in the text on this page three times. The caption should also be theta(t) instead of y(t).


Section 4.3, Page 391
The three references to y(0) in the second paragraph should be x(0) .


Section 4.4, Page 400
The characteristic polynomial is incorrect, it should read:

-(lamda^3 + (7/3)lamda^2 +(20/9)lamda + 8/9)
.
This will make the eigenvalues exactly -1, -2/3 + (2/3)i and -2/3 - (2/3)i.


Section 5.1, Page 421, Exercise 9.
The particular solution has a sign off it should be

y(t)=+(e^-t)cos(2t)/40 - 2(e^-t)sin(2t)/40 +37((e^-t)(2cos(2t)+sin(2t))/40 + (e^t)/8


Section 5.3, Page 440, Exercise 7.
The energy function should be

H(y,v) = 0.5 v^2 + 2 y^2


Section 6.1, Page 513, Lab 1.
The function involved is

f(x) = x^3 -3x^2 + 2x = x(x-1)(x-2)


Section 7.1, Page 527, Exercise 1a.
The answer in the back of the book is wrong. The correct answer is

y(t) = 1/(1 + t^2).

Don't know how this error crept into the book since the right answer appears several times in the section!


Section 7.3, Page 543
In the TI graphing calculator program the first line should read:

: (B - A) / N -> H


Section 7.4, Page 551, Exercise 3.
The answer in the back of the book is wrong. The answer stated there is for the t-interval [0, 1], not [0, 2]. The correct answer is

That should get you going in the right direction!


Section 8.2, Page 575.
There is a typo in the final solution of the example presented. The correct solution is:

y(t)=u_5(t)(sin(t-5)-1/sqrt{3}sin(sqrt{3}(t-5)))+cos(sqrt{3}t)


Section 8.2, Page 577, Exercise 17.
There is a typo in the answer in the back of the book. The correct answer is:

L[z] = (1/s^2) - e^{-s}/(s(1-e^{-s}))

While we have made every effort to eliminate errors in this preliminary edition of the text, undoubtedly my coauthors missed a few. If you do find an error or other infelicity, please let us know by clicking here and sending e-mail to odes@math.bu.edu. We also appreciate hearing any comments you might like to make about the text.


Special thanks to David Arnold, Brian Bradie, David Brown, Richard J. Charnigo, Jr., El wood Devaney, David Dudley, Greg Fredericks, Stephen Hrutka, Mike Hurley, Andres Martinez, Alejandro Montes, Kent Morrison, Steve Pennell, Avijit Purkayastha, Peggy Rejto, Phillip Straffin and Bob Worth for pointing out errors to us.

Last update: December 8, 1996.